Given two exponential random variables $X_1$ and $X_2$ and their expected values, how do i approach to find the expected value of $Y = min(X_1, X_2)$

The name of the pictureThe name of the pictureThe name of the pictureClash Royale CLAN TAG#URR8PPP











up vote
1
down vote

favorite
1












Here's the problem statement :



``Let $X_1$ and $X_2$ be two exponentially distributed random variables with mean $0.5$ and $0.25$. What is the mean of $Y = min(X_1, X_2)$? ''



We know, the probability density function of exponential function is as follows :



$f(x) = lambda e^-lambda x$ ; $x geq 0$ and $0$ everywhere else.



Also, $E(x) = frac1lambda$ and hence $lambda_1 = frac10.5 = 2$ and $lambda_2 = frac10.25 = 4$



The official solution is as follows :



$$E(Y) = frac1lambda_1 + lambda_2 = frac12 + 4 = frac 16$$



Please help me understand the solution !



Thank you so much.







share|cite|improve this question





















  • @ZacharySelk its the pdf of exponential distribution $f(x) = lambda e^-lambda x ; x geq 0$ and $0$ everywhere else
    – bluespring
    Jul 29 at 23:35










  • The official solution looks sensible. One illustration is that you arrive at a bus stop served by two routes, one with a service of $2$ buses an hour and the other with a service of $4$ buses an hour, the two routes being independent Poisson processes, so the expected time until a bus on the first route is $frac12$ hour and the expected time until a bus on the second route is $frac14$ hour. You are only going a short distance where the routes coincide, so do not care which arrives first. For you the rate is $6$ buses an hour and the expected time until the first bus is $frac16$ hour
    – Henry
    Jul 30 at 0:14











  • @Henry Perhaps you're right ! i was however expecting a little conventional approach. your method reminds me of queuing theory, where the arrival distribution is poisson and the service distribution is exponential. However, you've assumed arrival to be exponential here. Again, i'm not very good at maths, so pardon me.
    – bluespring
    Jul 30 at 0:29














up vote
1
down vote

favorite
1












Here's the problem statement :



``Let $X_1$ and $X_2$ be two exponentially distributed random variables with mean $0.5$ and $0.25$. What is the mean of $Y = min(X_1, X_2)$? ''



We know, the probability density function of exponential function is as follows :



$f(x) = lambda e^-lambda x$ ; $x geq 0$ and $0$ everywhere else.



Also, $E(x) = frac1lambda$ and hence $lambda_1 = frac10.5 = 2$ and $lambda_2 = frac10.25 = 4$



The official solution is as follows :



$$E(Y) = frac1lambda_1 + lambda_2 = frac12 + 4 = frac 16$$



Please help me understand the solution !



Thank you so much.







share|cite|improve this question





















  • @ZacharySelk its the pdf of exponential distribution $f(x) = lambda e^-lambda x ; x geq 0$ and $0$ everywhere else
    – bluespring
    Jul 29 at 23:35










  • The official solution looks sensible. One illustration is that you arrive at a bus stop served by two routes, one with a service of $2$ buses an hour and the other with a service of $4$ buses an hour, the two routes being independent Poisson processes, so the expected time until a bus on the first route is $frac12$ hour and the expected time until a bus on the second route is $frac14$ hour. You are only going a short distance where the routes coincide, so do not care which arrives first. For you the rate is $6$ buses an hour and the expected time until the first bus is $frac16$ hour
    – Henry
    Jul 30 at 0:14











  • @Henry Perhaps you're right ! i was however expecting a little conventional approach. your method reminds me of queuing theory, where the arrival distribution is poisson and the service distribution is exponential. However, you've assumed arrival to be exponential here. Again, i'm not very good at maths, so pardon me.
    – bluespring
    Jul 30 at 0:29












up vote
1
down vote

favorite
1









up vote
1
down vote

favorite
1






1





Here's the problem statement :



``Let $X_1$ and $X_2$ be two exponentially distributed random variables with mean $0.5$ and $0.25$. What is the mean of $Y = min(X_1, X_2)$? ''



We know, the probability density function of exponential function is as follows :



$f(x) = lambda e^-lambda x$ ; $x geq 0$ and $0$ everywhere else.



Also, $E(x) = frac1lambda$ and hence $lambda_1 = frac10.5 = 2$ and $lambda_2 = frac10.25 = 4$



The official solution is as follows :



$$E(Y) = frac1lambda_1 + lambda_2 = frac12 + 4 = frac 16$$



Please help me understand the solution !



Thank you so much.







share|cite|improve this question













Here's the problem statement :



``Let $X_1$ and $X_2$ be two exponentially distributed random variables with mean $0.5$ and $0.25$. What is the mean of $Y = min(X_1, X_2)$? ''



We know, the probability density function of exponential function is as follows :



$f(x) = lambda e^-lambda x$ ; $x geq 0$ and $0$ everywhere else.



Also, $E(x) = frac1lambda$ and hence $lambda_1 = frac10.5 = 2$ and $lambda_2 = frac10.25 = 4$



The official solution is as follows :



$$E(Y) = frac1lambda_1 + lambda_2 = frac12 + 4 = frac 16$$



Please help me understand the solution !



Thank you so much.









share|cite|improve this question












share|cite|improve this question




share|cite|improve this question








edited Jul 30 at 23:49
























asked Jul 29 at 23:22









bluespring

85




85











  • @ZacharySelk its the pdf of exponential distribution $f(x) = lambda e^-lambda x ; x geq 0$ and $0$ everywhere else
    – bluespring
    Jul 29 at 23:35










  • The official solution looks sensible. One illustration is that you arrive at a bus stop served by two routes, one with a service of $2$ buses an hour and the other with a service of $4$ buses an hour, the two routes being independent Poisson processes, so the expected time until a bus on the first route is $frac12$ hour and the expected time until a bus on the second route is $frac14$ hour. You are only going a short distance where the routes coincide, so do not care which arrives first. For you the rate is $6$ buses an hour and the expected time until the first bus is $frac16$ hour
    – Henry
    Jul 30 at 0:14











  • @Henry Perhaps you're right ! i was however expecting a little conventional approach. your method reminds me of queuing theory, where the arrival distribution is poisson and the service distribution is exponential. However, you've assumed arrival to be exponential here. Again, i'm not very good at maths, so pardon me.
    – bluespring
    Jul 30 at 0:29
















  • @ZacharySelk its the pdf of exponential distribution $f(x) = lambda e^-lambda x ; x geq 0$ and $0$ everywhere else
    – bluespring
    Jul 29 at 23:35










  • The official solution looks sensible. One illustration is that you arrive at a bus stop served by two routes, one with a service of $2$ buses an hour and the other with a service of $4$ buses an hour, the two routes being independent Poisson processes, so the expected time until a bus on the first route is $frac12$ hour and the expected time until a bus on the second route is $frac14$ hour. You are only going a short distance where the routes coincide, so do not care which arrives first. For you the rate is $6$ buses an hour and the expected time until the first bus is $frac16$ hour
    – Henry
    Jul 30 at 0:14











  • @Henry Perhaps you're right ! i was however expecting a little conventional approach. your method reminds me of queuing theory, where the arrival distribution is poisson and the service distribution is exponential. However, you've assumed arrival to be exponential here. Again, i'm not very good at maths, so pardon me.
    – bluespring
    Jul 30 at 0:29















@ZacharySelk its the pdf of exponential distribution $f(x) = lambda e^-lambda x ; x geq 0$ and $0$ everywhere else
– bluespring
Jul 29 at 23:35




@ZacharySelk its the pdf of exponential distribution $f(x) = lambda e^-lambda x ; x geq 0$ and $0$ everywhere else
– bluespring
Jul 29 at 23:35












The official solution looks sensible. One illustration is that you arrive at a bus stop served by two routes, one with a service of $2$ buses an hour and the other with a service of $4$ buses an hour, the two routes being independent Poisson processes, so the expected time until a bus on the first route is $frac12$ hour and the expected time until a bus on the second route is $frac14$ hour. You are only going a short distance where the routes coincide, so do not care which arrives first. For you the rate is $6$ buses an hour and the expected time until the first bus is $frac16$ hour
– Henry
Jul 30 at 0:14





The official solution looks sensible. One illustration is that you arrive at a bus stop served by two routes, one with a service of $2$ buses an hour and the other with a service of $4$ buses an hour, the two routes being independent Poisson processes, so the expected time until a bus on the first route is $frac12$ hour and the expected time until a bus on the second route is $frac14$ hour. You are only going a short distance where the routes coincide, so do not care which arrives first. For you the rate is $6$ buses an hour and the expected time until the first bus is $frac16$ hour
– Henry
Jul 30 at 0:14













@Henry Perhaps you're right ! i was however expecting a little conventional approach. your method reminds me of queuing theory, where the arrival distribution is poisson and the service distribution is exponential. However, you've assumed arrival to be exponential here. Again, i'm not very good at maths, so pardon me.
– bluespring
Jul 30 at 0:29




@Henry Perhaps you're right ! i was however expecting a little conventional approach. your method reminds me of queuing theory, where the arrival distribution is poisson and the service distribution is exponential. However, you've assumed arrival to be exponential here. Again, i'm not very good at maths, so pardon me.
– bluespring
Jul 30 at 0:29










3 Answers
3






active

oldest

votes

















up vote
1
down vote



accepted










$$E[Y] = int_0^inftyy,f_Y(y),dy$$



$$f_Y(y)=fracdF_Y(y)dy$$



Assuming $X_1$ and $X_2$ are independent, we have



$$F_Y(y)=textPleft[min(X_1, X_2)leq yright] \= 1- textPleft[min(X_1, X_2) > yright] \= 1-textPleft[X_1 > yright]textPleft[X_2 > yright]\=1-left[1-textPleft[X_1leq yright]right]left[1-textPleft[X_2leq yright]right]\=1-expleft(-y[lambda_1+lambda_2]right)$$



So



$$f_Y(y) = (lambda_1+lambda_2)expleft(-y[lambda_1+lambda_2]right)$$



Substitute in the first integration for the expectation and do the integration.






share|cite|improve this answer























  • @bluespring $$1-left[1-textPleft[X_1leq yright]right]left[1-textPleft[X_2leq yright]right] = 1-left[1-left1-exp(-lambda_1,y)rightright]left[1-left1-exp(-lambda_2,y)rightright]=1-expleft(-y[lambda_1+lambda_2]right)$$
    – BlackMath
    Jul 30 at 2:04











  • @bluespring If there is a simpler way, I would like to hear it, too. Simpler is better. Is there something that you find difficult to understand in my answer? I will be glad to explain more.
    – BlackMath
    Jul 30 at 3:55


















up vote
1
down vote













$f(X_2)<f(X_1)$ is wrong. The correct method is as follows: $PY>t=PX_1>tPX_2>t=int_t^infty (0.5) e^-(0.5)s ds int_t^infty (0.25) e^-(0.25)s ds$. Compute the integrals and differentiate to get the density of $Y$. Then compute its expectation.






share|cite|improve this answer























  • @ZacharySelk Thanks for the correction.
    – Kavi Rama Murthy
    Jul 29 at 23:39










  • Thank you sir for your kind help. I realize my error now.
    – bluespring
    Jul 30 at 23:39

















up vote
1
down vote













If $X_1,X_2$ are independent, it is explicitly possible that the event of $X_1<X_2$ may occur.



The fact that the probability denisity (or cumulative distribution) function of $X_2$ is greater than that of $X_1$ at every point does not mean $X_2$ will always have the lesser value; rather it suggests that it is more likely to.



Indeed $mathsf P(X_1leqslant X_2)=int_0^inftyint_0^t 8 e^- 2s-4tmathsf d smathsf d t=tfrac 13$






share|cite|improve this answer























  • This is some seriously advanced maths. I'm not able to follow a lot here. I'm from the engineering background, so we haven't learnt all this.
    – bluespring
    Jul 29 at 23:53










  • but thank you really for your response, its very much appreciated !
    – bluespring
    Jul 30 at 0:00










Your Answer




StackExchange.ifUsing("editor", function ()
return StackExchange.using("mathjaxEditing", function ()
StackExchange.MarkdownEditor.creationCallbacks.add(function (editor, postfix)
StackExchange.mathjaxEditing.prepareWmdForMathJax(editor, postfix, [["$", "$"], ["\\(","\\)"]]);
);
);
, "mathjax-editing");

StackExchange.ready(function()
var channelOptions =
tags: "".split(" "),
id: "69"
;
initTagRenderer("".split(" "), "".split(" "), channelOptions);

StackExchange.using("externalEditor", function()
// Have to fire editor after snippets, if snippets enabled
if (StackExchange.settings.snippets.snippetsEnabled)
StackExchange.using("snippets", function()
createEditor();
);

else
createEditor();

);

function createEditor()
StackExchange.prepareEditor(
heartbeatType: 'answer',
convertImagesToLinks: true,
noModals: false,
showLowRepImageUploadWarning: true,
reputationToPostImages: 10,
bindNavPrevention: true,
postfix: "",
noCode: true, onDemand: true,
discardSelector: ".discard-answer"
,immediatelyShowMarkdownHelp:true
);



);








 

draft saved


draft discarded


















StackExchange.ready(
function ()
StackExchange.openid.initPostLogin('.new-post-login', 'https%3a%2f%2fmath.stackexchange.com%2fquestions%2f2866522%2fgiven-two-exponential-random-variables-x-1-and-x-2-and-their-expected-values%23new-answer', 'question_page');

);

Post as a guest






























3 Answers
3






active

oldest

votes








3 Answers
3






active

oldest

votes









active

oldest

votes






active

oldest

votes








up vote
1
down vote



accepted










$$E[Y] = int_0^inftyy,f_Y(y),dy$$



$$f_Y(y)=fracdF_Y(y)dy$$



Assuming $X_1$ and $X_2$ are independent, we have



$$F_Y(y)=textPleft[min(X_1, X_2)leq yright] \= 1- textPleft[min(X_1, X_2) > yright] \= 1-textPleft[X_1 > yright]textPleft[X_2 > yright]\=1-left[1-textPleft[X_1leq yright]right]left[1-textPleft[X_2leq yright]right]\=1-expleft(-y[lambda_1+lambda_2]right)$$



So



$$f_Y(y) = (lambda_1+lambda_2)expleft(-y[lambda_1+lambda_2]right)$$



Substitute in the first integration for the expectation and do the integration.






share|cite|improve this answer























  • @bluespring $$1-left[1-textPleft[X_1leq yright]right]left[1-textPleft[X_2leq yright]right] = 1-left[1-left1-exp(-lambda_1,y)rightright]left[1-left1-exp(-lambda_2,y)rightright]=1-expleft(-y[lambda_1+lambda_2]right)$$
    – BlackMath
    Jul 30 at 2:04











  • @bluespring If there is a simpler way, I would like to hear it, too. Simpler is better. Is there something that you find difficult to understand in my answer? I will be glad to explain more.
    – BlackMath
    Jul 30 at 3:55















up vote
1
down vote



accepted










$$E[Y] = int_0^inftyy,f_Y(y),dy$$



$$f_Y(y)=fracdF_Y(y)dy$$



Assuming $X_1$ and $X_2$ are independent, we have



$$F_Y(y)=textPleft[min(X_1, X_2)leq yright] \= 1- textPleft[min(X_1, X_2) > yright] \= 1-textPleft[X_1 > yright]textPleft[X_2 > yright]\=1-left[1-textPleft[X_1leq yright]right]left[1-textPleft[X_2leq yright]right]\=1-expleft(-y[lambda_1+lambda_2]right)$$



So



$$f_Y(y) = (lambda_1+lambda_2)expleft(-y[lambda_1+lambda_2]right)$$



Substitute in the first integration for the expectation and do the integration.






share|cite|improve this answer























  • @bluespring $$1-left[1-textPleft[X_1leq yright]right]left[1-textPleft[X_2leq yright]right] = 1-left[1-left1-exp(-lambda_1,y)rightright]left[1-left1-exp(-lambda_2,y)rightright]=1-expleft(-y[lambda_1+lambda_2]right)$$
    – BlackMath
    Jul 30 at 2:04











  • @bluespring If there is a simpler way, I would like to hear it, too. Simpler is better. Is there something that you find difficult to understand in my answer? I will be glad to explain more.
    – BlackMath
    Jul 30 at 3:55













up vote
1
down vote



accepted







up vote
1
down vote



accepted






$$E[Y] = int_0^inftyy,f_Y(y),dy$$



$$f_Y(y)=fracdF_Y(y)dy$$



Assuming $X_1$ and $X_2$ are independent, we have



$$F_Y(y)=textPleft[min(X_1, X_2)leq yright] \= 1- textPleft[min(X_1, X_2) > yright] \= 1-textPleft[X_1 > yright]textPleft[X_2 > yright]\=1-left[1-textPleft[X_1leq yright]right]left[1-textPleft[X_2leq yright]right]\=1-expleft(-y[lambda_1+lambda_2]right)$$



So



$$f_Y(y) = (lambda_1+lambda_2)expleft(-y[lambda_1+lambda_2]right)$$



Substitute in the first integration for the expectation and do the integration.






share|cite|improve this answer















$$E[Y] = int_0^inftyy,f_Y(y),dy$$



$$f_Y(y)=fracdF_Y(y)dy$$



Assuming $X_1$ and $X_2$ are independent, we have



$$F_Y(y)=textPleft[min(X_1, X_2)leq yright] \= 1- textPleft[min(X_1, X_2) > yright] \= 1-textPleft[X_1 > yright]textPleft[X_2 > yright]\=1-left[1-textPleft[X_1leq yright]right]left[1-textPleft[X_2leq yright]right]\=1-expleft(-y[lambda_1+lambda_2]right)$$



So



$$f_Y(y) = (lambda_1+lambda_2)expleft(-y[lambda_1+lambda_2]right)$$



Substitute in the first integration for the expectation and do the integration.







share|cite|improve this answer















share|cite|improve this answer



share|cite|improve this answer








edited Jul 30 at 2:06


























answered Jul 30 at 1:07









BlackMath

877




877











  • @bluespring $$1-left[1-textPleft[X_1leq yright]right]left[1-textPleft[X_2leq yright]right] = 1-left[1-left1-exp(-lambda_1,y)rightright]left[1-left1-exp(-lambda_2,y)rightright]=1-expleft(-y[lambda_1+lambda_2]right)$$
    – BlackMath
    Jul 30 at 2:04











  • @bluespring If there is a simpler way, I would like to hear it, too. Simpler is better. Is there something that you find difficult to understand in my answer? I will be glad to explain more.
    – BlackMath
    Jul 30 at 3:55

















  • @bluespring $$1-left[1-textPleft[X_1leq yright]right]left[1-textPleft[X_2leq yright]right] = 1-left[1-left1-exp(-lambda_1,y)rightright]left[1-left1-exp(-lambda_2,y)rightright]=1-expleft(-y[lambda_1+lambda_2]right)$$
    – BlackMath
    Jul 30 at 2:04











  • @bluespring If there is a simpler way, I would like to hear it, too. Simpler is better. Is there something that you find difficult to understand in my answer? I will be glad to explain more.
    – BlackMath
    Jul 30 at 3:55
















@bluespring $$1-left[1-textPleft[X_1leq yright]right]left[1-textPleft[X_2leq yright]right] = 1-left[1-left1-exp(-lambda_1,y)rightright]left[1-left1-exp(-lambda_2,y)rightright]=1-expleft(-y[lambda_1+lambda_2]right)$$
– BlackMath
Jul 30 at 2:04





@bluespring $$1-left[1-textPleft[X_1leq yright]right]left[1-textPleft[X_2leq yright]right] = 1-left[1-left1-exp(-lambda_1,y)rightright]left[1-left1-exp(-lambda_2,y)rightright]=1-expleft(-y[lambda_1+lambda_2]right)$$
– BlackMath
Jul 30 at 2:04













@bluespring If there is a simpler way, I would like to hear it, too. Simpler is better. Is there something that you find difficult to understand in my answer? I will be glad to explain more.
– BlackMath
Jul 30 at 3:55





@bluespring If there is a simpler way, I would like to hear it, too. Simpler is better. Is there something that you find difficult to understand in my answer? I will be glad to explain more.
– BlackMath
Jul 30 at 3:55











up vote
1
down vote













$f(X_2)<f(X_1)$ is wrong. The correct method is as follows: $PY>t=PX_1>tPX_2>t=int_t^infty (0.5) e^-(0.5)s ds int_t^infty (0.25) e^-(0.25)s ds$. Compute the integrals and differentiate to get the density of $Y$. Then compute its expectation.






share|cite|improve this answer























  • @ZacharySelk Thanks for the correction.
    – Kavi Rama Murthy
    Jul 29 at 23:39










  • Thank you sir for your kind help. I realize my error now.
    – bluespring
    Jul 30 at 23:39














up vote
1
down vote













$f(X_2)<f(X_1)$ is wrong. The correct method is as follows: $PY>t=PX_1>tPX_2>t=int_t^infty (0.5) e^-(0.5)s ds int_t^infty (0.25) e^-(0.25)s ds$. Compute the integrals and differentiate to get the density of $Y$. Then compute its expectation.






share|cite|improve this answer























  • @ZacharySelk Thanks for the correction.
    – Kavi Rama Murthy
    Jul 29 at 23:39










  • Thank you sir for your kind help. I realize my error now.
    – bluespring
    Jul 30 at 23:39












up vote
1
down vote










up vote
1
down vote









$f(X_2)<f(X_1)$ is wrong. The correct method is as follows: $PY>t=PX_1>tPX_2>t=int_t^infty (0.5) e^-(0.5)s ds int_t^infty (0.25) e^-(0.25)s ds$. Compute the integrals and differentiate to get the density of $Y$. Then compute its expectation.






share|cite|improve this answer















$f(X_2)<f(X_1)$ is wrong. The correct method is as follows: $PY>t=PX_1>tPX_2>t=int_t^infty (0.5) e^-(0.5)s ds int_t^infty (0.25) e^-(0.25)s ds$. Compute the integrals and differentiate to get the density of $Y$. Then compute its expectation.







share|cite|improve this answer















share|cite|improve this answer



share|cite|improve this answer








edited Jul 29 at 23:38


























answered Jul 29 at 23:36









Kavi Rama Murthy

19.7k2829




19.7k2829











  • @ZacharySelk Thanks for the correction.
    – Kavi Rama Murthy
    Jul 29 at 23:39










  • Thank you sir for your kind help. I realize my error now.
    – bluespring
    Jul 30 at 23:39
















  • @ZacharySelk Thanks for the correction.
    – Kavi Rama Murthy
    Jul 29 at 23:39










  • Thank you sir for your kind help. I realize my error now.
    – bluespring
    Jul 30 at 23:39















@ZacharySelk Thanks for the correction.
– Kavi Rama Murthy
Jul 29 at 23:39




@ZacharySelk Thanks for the correction.
– Kavi Rama Murthy
Jul 29 at 23:39












Thank you sir for your kind help. I realize my error now.
– bluespring
Jul 30 at 23:39




Thank you sir for your kind help. I realize my error now.
– bluespring
Jul 30 at 23:39










up vote
1
down vote













If $X_1,X_2$ are independent, it is explicitly possible that the event of $X_1<X_2$ may occur.



The fact that the probability denisity (or cumulative distribution) function of $X_2$ is greater than that of $X_1$ at every point does not mean $X_2$ will always have the lesser value; rather it suggests that it is more likely to.



Indeed $mathsf P(X_1leqslant X_2)=int_0^inftyint_0^t 8 e^- 2s-4tmathsf d smathsf d t=tfrac 13$






share|cite|improve this answer























  • This is some seriously advanced maths. I'm not able to follow a lot here. I'm from the engineering background, so we haven't learnt all this.
    – bluespring
    Jul 29 at 23:53










  • but thank you really for your response, its very much appreciated !
    – bluespring
    Jul 30 at 0:00














up vote
1
down vote













If $X_1,X_2$ are independent, it is explicitly possible that the event of $X_1<X_2$ may occur.



The fact that the probability denisity (or cumulative distribution) function of $X_2$ is greater than that of $X_1$ at every point does not mean $X_2$ will always have the lesser value; rather it suggests that it is more likely to.



Indeed $mathsf P(X_1leqslant X_2)=int_0^inftyint_0^t 8 e^- 2s-4tmathsf d smathsf d t=tfrac 13$






share|cite|improve this answer























  • This is some seriously advanced maths. I'm not able to follow a lot here. I'm from the engineering background, so we haven't learnt all this.
    – bluespring
    Jul 29 at 23:53










  • but thank you really for your response, its very much appreciated !
    – bluespring
    Jul 30 at 0:00












up vote
1
down vote










up vote
1
down vote









If $X_1,X_2$ are independent, it is explicitly possible that the event of $X_1<X_2$ may occur.



The fact that the probability denisity (or cumulative distribution) function of $X_2$ is greater than that of $X_1$ at every point does not mean $X_2$ will always have the lesser value; rather it suggests that it is more likely to.



Indeed $mathsf P(X_1leqslant X_2)=int_0^inftyint_0^t 8 e^- 2s-4tmathsf d smathsf d t=tfrac 13$






share|cite|improve this answer















If $X_1,X_2$ are independent, it is explicitly possible that the event of $X_1<X_2$ may occur.



The fact that the probability denisity (or cumulative distribution) function of $X_2$ is greater than that of $X_1$ at every point does not mean $X_2$ will always have the lesser value; rather it suggests that it is more likely to.



Indeed $mathsf P(X_1leqslant X_2)=int_0^inftyint_0^t 8 e^- 2s-4tmathsf d smathsf d t=tfrac 13$







share|cite|improve this answer















share|cite|improve this answer



share|cite|improve this answer








edited Jul 29 at 23:55


























answered Jul 29 at 23:51









Graham Kemp

80k43275




80k43275











  • This is some seriously advanced maths. I'm not able to follow a lot here. I'm from the engineering background, so we haven't learnt all this.
    – bluespring
    Jul 29 at 23:53










  • but thank you really for your response, its very much appreciated !
    – bluespring
    Jul 30 at 0:00
















  • This is some seriously advanced maths. I'm not able to follow a lot here. I'm from the engineering background, so we haven't learnt all this.
    – bluespring
    Jul 29 at 23:53










  • but thank you really for your response, its very much appreciated !
    – bluespring
    Jul 30 at 0:00















This is some seriously advanced maths. I'm not able to follow a lot here. I'm from the engineering background, so we haven't learnt all this.
– bluespring
Jul 29 at 23:53




This is some seriously advanced maths. I'm not able to follow a lot here. I'm from the engineering background, so we haven't learnt all this.
– bluespring
Jul 29 at 23:53












but thank you really for your response, its very much appreciated !
– bluespring
Jul 30 at 0:00




but thank you really for your response, its very much appreciated !
– bluespring
Jul 30 at 0:00












 

draft saved


draft discarded


























 


draft saved


draft discarded














StackExchange.ready(
function ()
StackExchange.openid.initPostLogin('.new-post-login', 'https%3a%2f%2fmath.stackexchange.com%2fquestions%2f2866522%2fgiven-two-exponential-random-variables-x-1-and-x-2-and-their-expected-values%23new-answer', 'question_page');

);

Post as a guest













































































Comments

Popular posts from this blog

Color the edges and diagonals of a regular polygon

Relationship between determinant of matrix and determinant of adjoint?

What is the equation of a 3D cone with generalised tilt?